School starts soon! Add problem solving to your schedule with our math, science, and/or contest classes!

G
Topic
First Poster
Last Poster
A legendary inequality.
Imanamiri   1
N 28 minutes ago by arqady
Source: Inequality Russia
Let \( x, y, z \in \mathbb{R} \) such that
\[
x^2 + y^2 + z^2 = 2.
\]Prove that
\[
x + y + z \leq xyz + 2.
\]
1 reply
Imanamiri
an hour ago
arqady
28 minutes ago
Minimum Area for Monochromatic Triangles
steven_zhang123   1
N an hour ago by sarjinius
Source: 2025 Hope League Test 2 P5
Given a positive integer \( n \), find the smallest real number \( S \) such that no matter how each integer point in the coordinate plane \( xOy \) is colored with one of \( n \) different colors, there always exist three non-collinear points \( A, B, C \) of the same color such that the area of \(\triangle ABC\) is at most \( S \).
Proposed by Li Tianqin
1 reply
steven_zhang123
Jul 24, 2025
sarjinius
an hour ago
IMO ShortList 2001, combinatorics problem 7
orl   29
N an hour ago by eg4334
Source: IMO ShortList 2001, combinatorics problem 7
A pile of $n$ pebbles is placed in a vertical column. This configuration is modified according to the following rules. A pebble can be moved if it is at the top of a column which contains at least two more pebbles than the column immediately to its right. (If there are no pebbles to the right, think of this as a column with 0 pebbles.) At each stage, choose a pebble from among those that can be moved (if there are any) and place it at the top of the column to its right. If no pebbles can be moved, the configuration is called a final configuration. For each $n$, show that, no matter what choices are made at each stage, the final configuration obtained is unique. Describe that configuration in terms of $n$.

IMO ShortList 2001, combinatorics problem 7, alternative
29 replies
orl
Sep 30, 2004
eg4334
an hour ago
Sum is composite
anantmudgal09   8
N 2 hours ago by L13832
Source: India Practice TST 2017 D2 P2
Let $a,b,c,d$ be pairwise distinct positive integers such that $$\frac{a}{a+b}+\frac{b}{b+c}+\frac{c}{c+d}+\frac{d}{d+a}$$is an integer. Prove that $a+b+c+d$ is not a prime number.
8 replies
anantmudgal09
Dec 9, 2017
L13832
2 hours ago
geO 98 [convex hexagon ABCDEF with B + D + F = 360°]
Maverick   30
N 2 hours ago by Kempu33334
Source: IMO Shortlist 1998 Geometry 6
Let $ABCDEF$ be a convex hexagon such that $\angle B+\angle D+\angle F=360^{\circ }$ and \[ \frac{AB}{BC} \cdot \frac{CD}{DE} \cdot \frac{EF}{FA} = 1.  \] Prove that \[ \frac{BC}{CA} \cdot \frac{AE}{EF} \cdot \frac{FD}{DB} = 1.  \]
30 replies
Maverick
Oct 1, 2003
Kempu33334
2 hours ago
the answer is 3
Aiden-1089   1
N 2 hours ago by EeEeRUT
Source: APMO 2025 Problem 4
Let $n \geq 3$ be an integer. There are $n$ cells on a circle, and each cell is assigned either $0$ or $1$. There is a rooster on one of these cells, and it repeats the following operation:

$\bullet$ If the rooster is on a cell assigned $0$, it changes the assigned number to $1$ and moves to the next cell counterclockwise.
$\bullet$ If the rooster is on a cell assigned $1$, it changes the assigned number to $0$ and moves to the cell after the next cell counterclockwise.

Prove that the following statement holds after sufficiently many operations:
If the rooster is on a cell $C$, then the rooster would go around the circle exactly three times, stopping again at $C$. Moreover, every cell would be assigned the same number as it was assigned right before the rooster went around the circle three times.
1 reply
Aiden-1089
4 hours ago
EeEeRUT
2 hours ago
I want a hero to solve this algebra problem.
Imanamiri   1
N 2 hours ago by AleMM
Source: Old Russian.
Let \( a, b, c > 0 \) such that \( abc = 2 \).
Prove the following inequality:
\[
a^3 + b^3 + c^3 \geq a\sqrt{b+c} + b\sqrt{c+a} + c\sqrt{a+b}
\]
1 reply
Imanamiri
3 hours ago
AleMM
2 hours ago
Solve the equation
Tip_pay   5
N 2 hours ago by Tip_pay
Is there a nice way to replace this equation?

$$\dfrac{2x}{x^2-4x+2}+\dfrac{3x}{x^2+x+2}+\dfrac{5}{4}=0$$
If there is no suitable replacement, then how can you solve the equation without raising it to the 4th degree?
5 replies
Tip_pay
Today at 9:05 AM
Tip_pay
2 hours ago
Inequality
SunnyEvan   8
N 2 hours ago by arqady
Source: Own
Let $ a,b,c>0 ,$ such that: $ abc=1 .$ Prove that :
$$ \sqrt{64a^2+225}+\sqrt{64b^2+225}+\sqrt{64c^2+225} \leq (7\sqrt3-4)(a+b+c)+21(3-\sqrt3) $$
8 replies
SunnyEvan
Tuesday at 11:45 PM
arqady
2 hours ago
winning strategy, writing words on a blackboard
parmenides51   1
N 2 hours ago by LeYohan
Source: 1st Mathematics Regional Olympiad of Mexico Northwest 2018 P2
Alicia and Bob take turns writing words on a blackboard.
The rules are as follows:
a) Any word that has been written cannot be rewritten.
b) A player can only write a permutation of the previous word, or can simply simply remove one letter (whatever you want) from the previous word.
c) The first person who cannot write another word loses.
If Alice starts by typing the word ''Olympics" and Bob's next turn, who, do you think, has a winning strategy and what is it?
1 reply
parmenides51
Sep 6, 2022
LeYohan
2 hours ago
Arithmetic Sequence of Products
GrantStar   20
N 3 hours ago by BS2012
Source: IMO Shortlist 2023 N4
Let $a_1, \dots, a_n, b_1, \dots, b_n$ be $2n$ positive integers such that the $n+1$ products
\[a_1 a_2 a_3 \cdots a_n, b_1 a_2 a_3 \cdots a_n, b_1 b_2 a_3 \cdots a_n, \dots, b_1 b_2 b_3 \cdots b_n\]form a strictly increasing arithmetic progression in that order. Determine the smallest possible integer that could be the common difference of such an arithmetic progression.
20 replies
GrantStar
Jul 17, 2024
BS2012
3 hours ago
Integer Polynomial with P(a)=b, P(b)=c, and P(c)=a
Brut3Forc3   30
N Jun 6, 2025 by megahertz13
Source: 1974 USAMO Problem 1
Let $ a,b,$ and $ c$ denote three distinct integers, and let $ P$ denote a polynomial having integer coefficients. Show that it is impossible that $ P(a) = b, P(b) = c,$ and $ P(c) = a$.
30 replies
Brut3Forc3
Mar 13, 2010
megahertz13
Jun 6, 2025
Integer Polynomial with P(a)=b, P(b)=c, and P(c)=a
G H J
G H BBookmark kLocked kLocked NReply
Source: 1974 USAMO Problem 1
The post below has been deleted. Click to close.
This post has been deleted. Click here to see post.
Brut3Forc3
1948 posts
#1 • 2 Y
Y by Mathlover_1, Adventure10
Let $ a,b,$ and $ c$ denote three distinct integers, and let $ P$ denote a polynomial having integer coefficients. Show that it is impossible that $ P(a) = b, P(b) = c,$ and $ P(c) = a$.
Z K Y
The post below has been deleted. Click to close.
This post has been deleted. Click here to see post.
Agr_94_Math
881 posts
#2 • 2 Y
Y by Adventure10, Mango247
$ a,b,c \in \mathbb{Z}$, and $ a,b,c$ are distinct.
Since we have $ P[\mathbb{Z}]$, suppose we have by the given conditions to be exisitng such that
$ a-b | b-c$
$ b-c | c-a$
$ c-a | a-b$.
As we are dealing over integers,
$ 2b\ge c+a$
$ 2c\ge a+b$
$ 2a\ge b+c$
THis however implies equaltiy holds everywhere and therefore contradiction.
Z K Y
The post below has been deleted. Click to close.
This post has been deleted. Click here to see post.
Zhero
2043 posts
#3 • 8 Y
Y by cycliccircle, TheThor, Adventure10, and 5 other users
I don't think that's directly implied from the divisibilities you've arrived at. What if $ a - b > 0$ and $ b - c < 0$?

What I did
Z K Y
The post below has been deleted. Click to close.
This post has been deleted. Click here to see post.
KWTLEO
5 posts
#4 • 3 Y
Y by Adventure10, Mango247, and 1 other user
Agr_94_Math wrote:
$ a,b,c \in \mathbb{Z}$, and $ a,b,c$ are distinct.
Since we have $ P[\mathbb{Z}]$, suppose we have by the given conditions to be exisitng such that
$ a - b | b - c$
$ b - c | c - a$
$ c - a | a - b$.
you can arrive the result immediately by using the fact that
If $ x|y$ and $ y|x$, then $ |x|=|y|$.
Z K Y
The post below has been deleted. Click to close.
This post has been deleted. Click here to see post.
yugrey
2326 posts
#5 • 2 Y
Y by rd123, Adventure10
Well, thinking about the constant term (let it be z), b=z mod a. Similarly, c= z mod b, a=z mod c. b=z mod a=z mod (z mod(a)) =z mod z-na for some n, which means c=na, and c is a multiple of a. Similarly, a is a multiple of b, and b is in turn a multiple of c, so it follows that they are all equal, as a=1b, b=1c, and c=1a, as otherwise all the numbers would be unequal to themselves.

Q.E.D.

P.S. Oh my god, did I just do a USAMO problem? I didn't even qualify for AIME!! :D :lol: :) But they're much harder nowadays. Also, by = I meant congruence, sorry, and I obviously used the parentheses for order. I think, however, that it's a good solution. Edit: If there is something flawed with my solution, or if it is correct, tell me. If it is correct, I'd love to add this brief solution to the AoPS wiki, but I don't know latex. :blush:
Z K Y
The post below has been deleted. Click to close.
This post has been deleted. Click here to see post.
yugrey
2326 posts
#6 • 2 Y
Y by Adventure10, Mango247
Oh, there is a flaw. A number that is zmod z-na isn't necessarily equal to z-na.
Z K Y
The post below has been deleted. Click to close.
This post has been deleted. Click here to see post.
facis
642 posts
#8 • 2 Y
Y by Adventure10, Mango247
funny. A very similar problem was just used this year for the University of Illinois Undergrad Contest. It was pretty much this problem plus also the case of $P(a)=b$ and $P(b)=a$.
Z K Y
The post below has been deleted. Click to close.
This post has been deleted. Click here to see post.
hrithikguy
1791 posts
#9 • 3 Y
Y by MRenkhtur, Adventure10, Mango247
Suppose that $a < b < c$. Then $|P(a) - P(c)| = |b-a|< |a-c|$, a contradiction.
Next, suppose that $b < a < c$. Then $|P(b) - P(c)| = |c - a| < |c-b|$, also a contradiction.

Note that these are both contradictions because $|a-c|$ divides $|P(a) - P(c)|$ and $|c-b|$ divides $|P(b) - P(c)|$.
Z K Y
The post below has been deleted. Click to close.
This post has been deleted. Click here to see post.
cobbler
2180 posts
#10 • 2 Y
Y by Adventure10 and 1 other user
Using the well-known fact that $(a-b)\mid P(a)-P(b)$ we get
  • $\frac{P(a)-P(b)}{a-b}=\frac{b-c}{a-b}=k_1$
  • $\frac{P(b)-P(c)}{b-c}=\frac{c-a}{b-c}=k_2$
  • $\frac{P(c)-P(a)}{c-a}=\frac{a-b}{c-a}=k_3$

where the $k_i$ are integral.

Multiplying the first and last equations we have that \[\left(\frac{P(a)-P(b)}{a-b}\right)\left(\frac{P(c)-P(a)}{c-a}\right)=\frac{b-c}{c-a}=k_1k_3.\] But $\frac{b-c}{c-a}=\frac{1}{k_2}$, so the only way for $k_2$ to be an integer is if $|k_2|=1$. Similarly we get that $|k_1|=|k_3|=1$. If one of $k_1,k_2,k_3$ is -1, WLOG say $k_1$, then $\tfrac{b-c}{a-b}=-1\Leftrightarrow{b-c=-a+b}\Leftrightarrow{a=c}$, contradiction. Thus, $k_1,k_2,k_3$ are all equal to 1, and plugging these in it follows that $2a=b-c$, $2b=c-a$ and $2c=a-b$. Solving this system yields $a=b=c=0$, but $a,b,c$ are distinct integers by definition, a contradiction.
Z K Y
The post below has been deleted. Click to close.
This post has been deleted. Click here to see post.
sayantanchakraborty
505 posts
#11 • 2 Y
Y by Adventure10, Mango247
These conditions imply that [a-b]<=[b-c]<=[c-a]<=[a-b],where [a]=modulus of a, so equality holds everywhere.
Thus the fact that a,b,c are distinct implies (a-b)=(b-c)=(c-a).Solving we get a=b=c. Contradiction!
Z K Y
The post below has been deleted. Click to close.
This post has been deleted. Click here to see post.
bobthesmartypants
4336 posts
#12 • 1 Y
Y by Adventure10
Agr_94_Math wrote:
$ a,b,c \in \mathbb{Z}$, and $ a,b,c$ are distinct.
Since we have $ P[\mathbb{Z}]$, suppose we have by the given conditions to be exisitng such that
$ a-b | b-c$
$ b-c | c-a$
$ c-a | a-b$.
As we are dealing over integers,
$ 2b\ge c+a$
$ 2c\ge a+b$
$ 2a\ge b+c$
THis however implies equaltiy holds everywhere and therefore contradiction.

How did he immediately arrive at the conclusion:

$ a-b | b-c$
$ b-c | c-a$
$ c-a | a-b$
Z K Y
The post below has been deleted. Click to close.
This post has been deleted. Click here to see post.
Abe27342
146 posts
#13 • 2 Y
Y by Adventure10, Mango247
It's well-known that $a-b|P(a)-P(b)$ for integers $a$ and $b$ and a polynomial $P$ with integral coefficients. To see why, write out the expression $P(a)-P(b)$ and then refactor it so you get a lot of terms of the form $a^k-b^k$ for some $k$. Then $a-b$ divides all of these terms, and $P(a)-P(b)$ is a linear combination of terms of this form, so $a-b|P(a)-P(b)$.
Z K Y
The post below has been deleted. Click to close.
This post has been deleted. Click here to see post.
tastymath75025
3223 posts
#14 • 2 Y
Y by Adventure10, Mango247
Here's what I did:
Since $\frac{a-b}{b-c}, \frac{b-c}{c-a}, \frac{c-a}{a-b}$ are all integers and their product is equal to $1$, all three of them are each either $1$ or $-1$.
At least one of the fractions equals 1, so WLOG $\frac{a-b}{b-c}=1$. THen either both of the remaining fractions equal 1 or both equal -1, so we examine the two cases. Both cases give us that $a=b=c$, contradiction.
Z K Y
The post below has been deleted. Click to close.
This post has been deleted. Click here to see post.
bobthesmartypants
4336 posts
#15 • 1 Y
Y by Adventure10
Okay I guess I wasn't thinking. I forgot that $P(a)=b$ and $P(b)=c$. :|
Z K Y
The post below has been deleted. Click to close.
This post has been deleted. Click here to see post.
Wave-Particle
3690 posts
#16 • 3 Y
Y by luka2002, Adventure10, Mango247
Here is a solution I wrote up a while ago:

Lemma 1: If $P(x)$ is a polynomial with integer coefficients then $a-b|P(a)-P(b)$ where $a,b$ are integers.

Proof: Let $P(x)=a_n*x^n+a_{n-1}x^{n-1}...+a_1x+a_0$. We have $P(a)=a_n*a^n+a_{n-1}a^{n-1}...+a_1a+a_0$ and $P(b)=a_nb^n+a_{n-1}b^{n-1}...a_1b+a_0$. So $P(a)-P(b)=(a^n-b^n)a_n+(a^{n-1}-b^{n-1})a_{n-1}+...+(a-b)a_1$. Notice that $a^k-b^k=(a-b)(a^{k-1}+a^{k-2}b+...b^{k-1})$ so it is easy to see $a-b|P(a)-P(b)$.

We will prove this by contradiction. By Lemma 1 we have $a-b|P(a)-P(b)$, $b-c|P(b)-P(c)$, and $c-a|P(c)-P(a)$. We know what $P(a), P(b),$ and $P(c)$ are as they were given in the problem so we substitute them in. Let $\frac{b-c}{a-b}=k$, $\frac{c-a}{b-c}=m$, and $\frac{a-b}{c-a}=t$ where $k,m,t$ are integers. Note that $kmt=1$ and because $k,m,t$ are integers it follows either all of them are $1$ or two of them are $-1$ and the other is $1$. Let's split this into two cases:

Case 1: $k=m=t=1$.

From here we have $b-c=a-b$, $c-a=b-c$, and $a-b=c-a$. This gives us that $a=b=c$ but the problem said $a,b,c$ are distinct so this case is ruled out.

Case 2: WLOG let $k=m=-1$ and $t=1$.

This means that $a=b=c$ again, meaning that it is impossible for $P(a)=b$, $P(b)=c$, and $P(c)=a$.
Z K Y
The post below has been deleted. Click to close.
This post has been deleted. Click here to see post.
strategos21
491 posts
#17 • 2 Y
Y by Adventure10, Mango247
solution
This post has been edited 1 time. Last edited by strategos21, Jun 25, 2018, 3:27 PM
Z K Y
The post below has been deleted. Click to close.
This post has been deleted. Click here to see post.
Jzhang21
308 posts
#18 • 2 Y
Y by Adventure10, Mango247
Assume for the sake of contradiction that is possible for unique integers $a,b,c$. Let $P(x)=d_1x^n+d_2x^{n-1}+\cdots+d_n.$ Note that $$b=P(a)=d_1a^n+d_2a^{n-1}+\cdots+d_n$$$$c=P(b)=d_1b^n+d_2b^{n-1}+\cdots+d_n$$$$a=P(c)=d_1c^n+d_2c^{n-1}+\cdots+d_n$$Subtracting the second from the first, third from second, and first from third gives
$$b-c=P(a)-P(b)=d_1(a^n-b^n)+d_2(a^{n-1}-b^{n-1})+\cdots+ d_{n-1}(a-b)$$$$c-a=P(b)-P(c)=d_1(b^n-c^n)+d_2(b^{n-1}-c^{n-1})+\cdots +d_{n-1}(b-c)$$$$a-b=P(c)-P(a)=d_1(c^n-a^n)+d_2(c^{n-1}-a^{n-1})+\cdots +d_{n-1}(c-a)$$By the RHS, note that $a-b\mid P(a)-P(b)$ so $\lvert{a-b}\rvert\leq\lvert{b-c}\rvert.$ Similarly, $\lvert{b-c}\rvert\leq\lvert{c-a}\rvert$ and $\lvert{c-a}\rvert\leq\lvert{a-b}\rvert.$ Hence, $\lvert{a-b}\rvert\leq\lvert{b-c}\rvert\leq\lvert{c-a}\rvert\leq\lvert{a-b}\rvert$ so $\lvert{a-b}\rvert=\lvert{b-c}\rvert=\lvert{c-a}\rvert.$ Assume WLOG that $a>b>c$ so $a-b=a-c$ and $b-c=c-a.$ From the first equation, we get $b=c$ and substituting this in the second gives $c=a.$ Hence, $a=b=c$, contradicting the uniqueness of $a,b,c.$ $\blacksquare$
Z K Y
The post below has been deleted. Click to close.
This post has been deleted. Click here to see post.
amar_04
1917 posts
#19 • 1 Y
Y by Adventure10
A bit different finishing.......

Like others did, I also used the well known fact that $x-y|P(x)-P(y)$. So, by this fact we get $$a-b|b-c$$$$b-c|c-a$$$$c-a|a-b$$. Now we will consider this into two cases when $a-b=b-c$, $b-c=c-a$ and and the other case will be when $a-b<b-c$, $b-c<c-a$ and $c-a<a-b$. So, by adding all the inequalities we get $a+b+c<a+b+c$ which is impossible. Now we will consider when $a-b=b-c$, $b-c=c-a$ and $c-a=a-b$. From this we get $a=b=c$, which is clearly not possible, as the question says that $a,b,c$ are distinct integers. Hence, not possible.
This post has been edited 3 times. Last edited by amar_04, Aug 18, 2019, 5:53 AM
Z K Y
The post below has been deleted. Click to close.
This post has been deleted. Click here to see post.
Pluto04
797 posts
#20 • 1 Y
Y by Adventure10
Indian MO 1986.
Z K Y
The post below has been deleted. Click to close.
This post has been deleted. Click here to see post.
AopsUser101
1750 posts
#21 • 3 Y
Y by vsamc, v4913, Adventure10
Let $P(x) = d_nx^n+d_{n-1}x^{n-1}+...+d_0$. We have that $$P(m)-P(n) = d_n(m^n-n^n)+d_{n-1}(m^{n-1}-n^{n-1})+...+d_1(m-n)$$, which is clearly divisible by $m-n$. Hence, $$(a-b)|(b-c)$$$$(b-c)|(c-a)$$$$(c-a)|(a-b).$$Therefore, $|a-b|=|b-c|=|c-a|$. Letting $x=a-b$, $y=b-c$, and $x+y=c-a$, we have that $|x|=|y|=|x+y|$. $x=-y \implies a=c$, so we know that $x=y$. In other words, $|y|=|2y| \implies y = 0$. If $y=0$, then $b=c$, which is impossible. $\blacksquare$
This post has been edited 2 times. Last edited by AopsUser101, Feb 3, 2020, 12:38 AM
Z K Y
The post below has been deleted. Click to close.
This post has been deleted. Click here to see post.
Math-Shinai
396 posts
#22 • 1 Y
Y by Adventure10
I don't see the point of commenting on an ancient post that is well-known... especially when the solution that you just posted is basically the same as everyone else's
Z K Y
The post below has been deleted. Click to close.
This post has been deleted. Click here to see post.
vsamc
3814 posts
#23 • 2 Y
Y by Mango247, Mango247
@above well writing your solution up helps you become better at math because you can reflect on your solution.

my first ever USAMO solve :)
My Solution
This post has been edited 1 time. Last edited by vsamc, Jun 4, 2020, 4:32 PM
Z K Y
The post below has been deleted. Click to close.
This post has been deleted. Click here to see post.
bryanguo
1032 posts
#24 • 4 Y
Y by v4913, Mango247, Mango247, Mango247
My solution is basically @sayantanchakraborty's, but I feel like the explanation of mine is slightly more concise:

Solu
This post has been edited 1 time. Last edited by bryanguo, Jun 4, 2020, 10:18 PM
Z K Y
The post below has been deleted. Click to close.
This post has been deleted. Click here to see post.
OlympusHero
17020 posts
#25 • 1 Y
Y by Mango247
Solution
Z K Y
The post below has been deleted. Click to close.
This post has been deleted. Click here to see post.
Mogmog8
1080 posts
#26 • 2 Y
Y by centslordm, Mango247
Solution
Z K Y
The post below has been deleted. Click to close.
This post has been deleted. Click here to see post.
Sprites
478 posts
#27
Y by
FTSOC,assume that we can have $P(a)=b,P(b)=c,P(c)=a$ simultaneously.
Then $$a-c|P(P(c))-P(P(a))=a-b$$and $$a-b|P(P(b))-P(P(a))=a-c$$implying $b=c$, contradiction.
Z K Y
The post below has been deleted. Click to close.
This post has been deleted. Click here to see post.
lifeismathematics
1188 posts
#28
Y by
div and done
This post has been edited 1 time. Last edited by lifeismathematics, Nov 21, 2022, 1:22 PM
Z K Y
The post below has been deleted. Click to close.
This post has been deleted. Click here to see post.
huashiliao2020
1292 posts
#29
Y by
Agr_94_Math wrote:
$ a,b,c \in \mathbb{Z}$, and $ a,b,c$ are distinct.
Since we have $ P[\mathbb{Z}]$, suppose we have by the given conditions to be exisitng such that
$ a-b | b-c$
$ b-c | c-a$
$ c-a | a-b$.
As we are dealing over integers,
$ 2b\ge c+a$
$ 2c\ge a+b$
$ 2a\ge b+c$
THis however implies equaltiy holds everywhere and therefore contradiction.

hmm, I don't understand how you got to a-b|b-c. Can someone explain the omitted part?
Z K Y
The post below has been deleted. Click to close.
This post has been deleted. Click here to see post.
pco
23515 posts
#30
Y by
huashiliao2020 wrote:
hmm, I don't understand how you got to a-b|b-c. Can someone explain the omitted part?
$a-b|P(a)-P(b)=b-c$
Z K Y
The post below has been deleted. Click to close.
This post has been deleted. Click here to see post.
ryanbear
1058 posts
#31
Y by
$a-b|P(a)-P(b)$ --> $a-b|b-c$
Similarly, $b-c|c-a$, $c-a|a-b$
So this means $|a-b|=|b-c|=|c-a|$
WLOG $a<b<c$
$b-a=c-b=c-a$
$a=b$ (contradiction)
Z K Y
The post below has been deleted. Click to close.
This post has been deleted. Click here to see post.
megahertz13
3206 posts
#32
Y by
First, observe that \[P(a)-P(b)=P(b+(a-b))-P(b)\]is always a multiple of $b-a$. From this we obtain $a-b|b-c$, $b-c|c-a$, and $c-a|a-b$. Thus, \[|a-b|=|b-c|=|c-a|.\]Clearly $a-b=-(b-c)$ cannot hold, as that would imply $a=c$. Therefore, $a-b=b-c\implies c+a=2b$. Similarly, we get $a+b=2c$. Subtracting this gives $c-b=2b-2c\implies c=b$, a contradiction.
Z K Y
N Quick Reply
G
H
=
a